BM EM II

अब Quizwiz के साथ अपने होमवर्क और परीक्षाओं को एस करें!

A 75-year-old woman is brought to your office by her daughter. She lives alone since her husband died 5 years ago due to lung cancer. Her daughter tells you that she is having difficulty in doing housework for the past 6 months. She is more aggressive nowadays and asks similar questions repeatedly. Twice she forgot her home address while returning from the super market, when she was taken to the home by her neighbor. She has had 2 motor vehicle accidents in the past 2 months. She is suffering from osteoarthritis and using ibuprofen for this. She is using salbutamol inhaler along with montelukast to control her 20-year history of asthma. She was operated on for appendicitis at age 15 and fibroadenoma was removed at age 22. Her sister is suffering from some sort of memory problem. She denies use of alcohol, tobacco, or illicit drugs.General physical exam is normal. Temperature 37°C, heart rate 70, respiratory rate 16, and blood pressure 110/70. Neurological exam reveals that the patient is alert and oriented. Her speech has frequent pauses with word-finding difficulty. She is able to register 3 items but unable to repeat them after 2 minutes. She doesn't know today's date; neither does she know her date of birth. She is unable to name the parts of the body and list of vegetables. She is unable to draw a clock and unable to draw or copy a cube. She is able to follow simple commands but finds difficult in completing complex task. She asks your name repeatedly during the interview. The patient's mini-mental status examination is 20/30. The patient's cranial nerve II through XII are intact and muscle strength and cutaneous sensation are normal. Deep tendon reflexes are normal. HEENT reveals PEERLA with normal oral mucosa; neck is supple without any masses or adenopathy. Lungs reveal normal vesicular breathing and heart shows regular rate and rhythm without any murmur. Examination of abdomen is normal.Results of lab test are shown is the table. hemoglobin13.8g/dlLeukocyte count5,000/mlSegmented neutrophils60%Platelet count355,000/mlserum glucose120mg/dlSodium140mmol/dlPotassium4.7mmol/dlChloride105mmol/dlBicarbonate22mmol/dlTSH3U/mlT3 Serum130ng/dl4 Serum6ng/dlVitamin B128Folic acid1.7mEq/l She is started with donepezil but her symptoms deteriorate further during the next 6 months. Question What medication should be given to this patient now?

Memantine (AD) There is no cure for Alzheimer's disease. 4 cholinesterase inhibitors, tacrine, donepezil, rivastigmine, and galantamine, are currently approved for use in AD by the U.S. Food and Drug Administration (FDA). In patients with moderate to advanced dementia, add memantine to a cholinesterase inhibitor, or use memantine alone in patients who do not tolerate or benefit from a cholinesterase inhibitor. Current treatments include pharmaceutical, psychosocial, and caregiving.

A 23-year-old man presents with an unspecified personality disorder; although his group and individual therapy sessions are going well, he wants to gain even more improvement with his diagnosis. He feels that his mood, self-image, and personal relationships have improved overall, but he specifically states he wants to have better anger management. When he becomes angry, for whatever reason, he still becomes extremely irritated, hostile, and even aggressive. Question What pharmaceutical therapy would be most beneficial for this patient?

Mood stabilizers such as lamotrigine and topiramate (Borderline)

What is a normal Rapid Eye Movement (REM) latency?

90 min A normal REM latency is 90 minutes after initiation of NREM sleep. Shortened REM latency periods occur with major depression and narcolepsy.

After ingestion, the peak plasma level of methamphetamine is reached in:

30 min

What is the age cutoff for the diagnosis of Enuresis?

5 Enuresis, the repeated voiding of urine into the bed or clothes, must occur at least twice a week for at least 3 consecutive months, in a child at least 5 years old (or of an equivalent developmental level).

Normal sleep is characterized by (a) _____ distinct sleep stage(s)

5 REM and NREM 1-4 (NREM 3 and 4 are deepest sleeps)

A deficit of which of the following neurotransmitters is most strongly associated with Alzheimer's disease?

ACh

An 18-year-old male high school dropout recently set a fire in his old school classroom because he was dared to do so by other students. He has been sent to you for evaluation. You interview him and find that he had a number of problems related to truancy and fighting in school; he has been found with liquor in his locker at school, and he always seems to feel that someone else is responsible for his having done something. When he was 10, the patient burned down a barn; last year, he and some friends threw rocks at passing cars on the expressway. Past medical history is non-contributory. He denies any recreational drug use. Question What is the most likely diagnosis?

Antisocial personality disorder

An 18-year-old girl presents with a 5-month history of feeling tired most of the time. She used to be a healthy athlete and won many competitions in the past. Her daily activities are quite restricted nowadays, and school performance is deteriorating, which worries her parents. She is very anxious and finds it difficult to cope with the presenting condition. She also complains of burning epigastric discomfort for which she takes over the counter medication. Her weight is 45 kg; height is 1.7 m, and her current body-mass index is 15.5 kg/m2. On detailed questioning, she tells you that she is preoccupied most of the time with her present body weight and she is also conscious about her body image. She feels cold even during the hot summer days. She exercises daily and runs almost 5 miles. She admits that she hesitates to eat due to fear of gaining weight. She also uses laxatives to control her body weight and diuretics to increase her performance in sports. She has bowel movements 2-3 times daily. Her past medical history is unremarkable. She has normal menstrual periods, and her last one occurred 2 weeks ago. She denies any suicidal ideation. Her sleep pattern is normal. She has 1 sexual partner, and she uses a barrier method of contraception. She smokes 1 pack of cigarettes daily.On physical examination, she appears well groomed and alert and in no obvious distress. Temperature is 37.0°C, heart rate 60/m, blood pressure 90/50 mm Hg, respiratory rate 16/m. Orthostatic blood pressure results in an increase in heart rate of 15 bpm and a drop in blood pressure of 5 mmHg. HEENT exam reveals PEERLA; neck is supple without masses or adenopathy. Skin is dry. Cardiac examination reveals normal rate and rhythm without any murmur, rub, or gallop. Chest is clear to auscultation bilaterally. Abdomen is soft and nontender and gut sounds are normal. The patient's cranial nerves II through XII are intact and muscle strength and cutaneous sensation are normal. Deep tendon reflexes are normal.Her labs show: Hemoglobin12.8 g/dLLeukocyte count7,000/mLPlatelet count250,000/mLUrea19 mg/dLCreatinine0.8 mg/dLsodium135 mmol/dLpotassium3.3 mmol/dLcalcium8 mg/dLbicarbonate30 mEq/LTSH2 µU/mLT4 serum8 µU/mLT3 serum (RIA)150 ng/dLPhosphorous4 mg/dLBeta-HCGnegativeUrine D/Rnormal Question What is the most likely diagnosis?

Anorexia nervosa (low BMI)

In performing mental status testing, recent memory is best tested by

Asking the patient to remember a list of three items for 3-5 minutes

An 8-year-old boy presents for evaluation of problems at school and at home. His parents report that he does not pay attention in class, he is frequently in trouble for being disruptive, and he often forgets to do his schoolwork. He has had similar problems since starting school (in kindergarten), but they are worsening. The teacher reports the patient often seems distracted. He rarely sits still at his desk, fidgets often, and blurts out comments without waiting his turn. Physical examination is remarkable for increased motor activity but is otherwise normal. Question What is the best intervention for this patient's condition?

Initiate stimulant meds

A 6-year-old boy is brought to the pediatrician by his parents with the complaint of recurrent nightmares. The parents report that for the past week the child has been waking up about 2 hours after being put to bed, screaming and unresponsive to all efforts by the parents to calm the child. Within a few minutes, the child falls back asleep and the next morning does not recall any scary dreams or even waking up the previous night. Question What is the most likely diagnosis?

Night Terrors

A 19-year old male is brought to the ER. He displays violent behavior and is combative. His pupils appear normal, but there is some muscle rigidity. Question What might the preliminary diagnosis be?

Phencyclidine intoxication

PCP is an example of a dissociative anesthetic that, when abused or otherwise taken improperly, can cause a type of substance related disorder. How would this disorder be properly classified?

Phencyclidine-related disorder

Dreaming can occur in several sleep stages, but is most common in which stage?

REM

A person with obstructive sleep apnea can have dozens of apneic episodes and awakenings each night. Most of these occur during

REM sleep

Cognitive functioning is frequently assessed via psychometric testing, but frequently a clinical interview is the method for initial screening. Patients who are suspected of having a cognitive deficit and who may be of advanced age should:

Receive an evaluation for hearing loss

A 16-year-old slightly overweight girl presents because she wants to lose weight. She read online that drugs for ADD can help with weight loss. She is unsure of the name of the drug she read about but wonders if she can use them as diet pills instead. Since she was told that drugs like meth have similar properties, she also wants to know if these are safe to take for weight loss. She is told to avoid them because they can become habit-forming and their effectiveness is short-lived. Question What is the mechanism of action of the drug she is referring to?

Releases dopamine and norepinephrine stores (amphetamines)

Which of the following medications is the most appropriate for symptomatic treatment of agitation in an elderly patient with dementia?

Risperidone

Adverse effects of L-dopa in the treatment of Parkinson's disease include all of the following EXCEPT Hallucinations Motor fluctuations Dyskinesias Tremor Confusion

Tremor

What is the mechanism of action of clonidine (Catapres)?

a2 agonist (can be used for opiate withdrawal)

The deepest of the sleep stages is Stage 4 and is characterized by which EEG wave pattern?

delta waves

What is the term that is defined as the feeling or conviction that one is male or female?

gender identity

An essential feature of Borderline Personality Disorder (BPD) is

instability of relationships

A 20-year-old man presents because he is feeling nervous, cannot sleep at night, has a loss of appetite, a low-grade fever, a runny nose, and stomach cramps. On physical exam, the hair on his arms is standing on end, he is sweating, and his pupils are dilated. He states that he is accustomed to taking a certain drug, but he has not been able to obtain it for the last 72 hours. Question From what type of drug is this patient most likely withdrawing?

opiate

Conduct disorder would be a better diagnosis than oppositional defiant disorder when what trait is present?

Aggression to animals

A 32-year-old man was found sleeping on a subway vent. Physical exam shows a blood pressure of 90/50 mm Hg and a pulse of 135/min. He has bilateral nystagmus and abducens palsies, but pupils and optic disks are normal. He is ataxic and cannot stand for longer than a few seconds. During the exam, he does not speak spontaneously and he sometimes falls asleep but he is easily aroused. His answers to questions are minimal and some questions are not answered at all. He is disoriented and consistently misidentifies the police officers and hospital personnel. A complete mental status exam is impossible because the patient is unable to cooperate fully. Question What is the most likely diagnosis?

Alcoholic encephalopathy (Wernicke's Syndrome)

Relapse for those who have quit smoking is most often cited as a result of:

Attempting to alleviate negative mood states

A woman talks incessantly about how important her work is. She is director of an advocacy program. It seems as if her world revolves around her work; she believes that without her at the helm of the program, "the little people would just never be able to organize to fight their cause." Her co-workers describe her as arrogant and attention seeking and she rarely feels the need to empathize with anyone. Question What personality disorder does she likely have?

Narcissistic

A 17-year-old girl presents with her mother because she has been severely underweight for the past 3 years and the mother recently witnessed her vomiting in the bathroom. The patient states she is fearful to gain any weight, as she does not want to become fat. She denies the use of any recreational drugs or alcohol and states that she does not take any medications. She has no known medical history. Her menstrual cycles are regular, and she is not sexually active. On physical examination, vital signs are within reference ranges. BMI is 17 kg/m2. Excoriations on the back of her hands and swollen parotid glands are noted. The rest of her physical examination is unremarkable. A complete blood cell count and complete metabolic profile are ordered and results are pending. Question What is considered the first-line treatment for this patient?

Psychodynamic psychotherapy (Anorexia, d/t low BMI) The DSM-5 criteria for diagnosing anorexia nervosa include inability to maintain a minimally normal weight, a devastating fear of weight gain, relentless dietary habits that prevent weight gain, and a disturbance in perception of body weight and shape. Amenorrhea is no longer a diagnostic criterion for this disorder. There are two subtypes of anorexia nervosa: restricting and purging. The main differentiation between anorexia nervosa and bulimia is patient BMI. Patients with anorexia are underweight and want to remain thin due to fear of gaining weight; patients with bulimia are of normal weight or sometimes overweight. Anorexia is treated using psychotherapy, which includes a multidisciplinary team of psychologists, psychiatrists, dietitians, and eating disorder experts. Psychodynamic psychotherapy such as family therapy and cognitive-based therapy are considered first-line treatments for anorexia nervosa. Estrogen replacement in the form of oral contraceptive pills has been used to treat osteopenia in patients with anorexia nervosa, as this is a common complication, but this treatment is not indicated solely for the treatment of anorexia. Fluoxetine is an SSRI that has not been shown effective in treating anorexia, but it has value in treating bulimia nervosa. Many patients with anorexia have mood disorders such as depression, and an SSRI can be beneficial. Psychotherapy with adjunctive low-dose olanzapine may be useful for anorexia nervosa inpatient treatment, but olanzapine alone is not used to treat anorexia. Individuals with anorexia nervosa at risk medically or psychiatrically require inpatient treatment. Indications for hospital admission: 85% or less of expected weight or <3rd percentile for BMI Lack of any weight gain Significant edema Physiologic decompensation: severe electrolyte disturbances, cardiac disturbances (EKG changes), altered mental status, severe orthostatic changes Temperature <36°C Pulse <45 beats per minute Psychosis or high risk of suicide Symptoms refractory to outpatient treatment

A 32-year-old woman lives alone, has no friends, dresses strangely, and talks in an unusual manner. Although odd, she is able to maintain employment and has not been known to have trouble with the law. Question What is the most likely diagnosis?

Schizotypal This patient's presentation is most consistent with schizotypal personality disorder, which is a condition characterized by a lifelong pattern of indifference to others as well as odd behavior and thinking, and/or unconventional beliefs, detachment, and the need for social isolation. Although isolated, patients are not usually unhappy with their lack of social contact.

The symptoms of dyslexia are believed to be caused by maldevelopment in a particular part of the brain, called the:

Thalamus Researchers think that the maldeveloped left thalamus is the neurological basis for why some dyslexic children cannot distinguish among fast consonant sounds (p, b, k, or t).

A distinguishing feature between Paranoid Personality Disorder and Schizotypal Personality Disorder is:

perceptual distortions

The rate-limiting enzyme in the synthesis of dopamine is

tyrosine

A 42-year-old man presents with difficulties getting to sleep. His wife has noted that the patient moves around almost continually at bedtime. The patient specifically describes crawling and tingling sensations on his lower limbs and sometimes his arms that are relieved by movement. He has no remarkable past medical history. He denies smoking, caffeine and alcohol. He admits to worrying about a major work contract that will be signed next month because, "My job is on the line", he says. His physical examination and neurological examination are normal. All lab work is normal. Question What is the most appropriate response to give to the patient?

"You have restless leg syndrome"

A 79-year-old Caucasian woman visits her family physician because she is concerned about her sleep. "I just don't sleep as well as I used to." On close questioning, the patient spends more time in bed and sleeps less soundly than she did in her youth. She does not drink alcohol or caffeine, and she takes acetaminophen occasionally for her osteoarthritis. She goes to bed at 11:00 PM, right after the evening news on television. She is asleep within 15 minutes. She wakes several times nightly, but falls back to sleep promptly. She does not snore, have abnormal movements, or experience other nocturnal problems. Her husband confirms this information. She feels refreshed in the morning and puts in a full day volunteering at a local hospital and participating in several civic organizations. Her physical examination is normal, other than mild degenerative changes in the hands and right knee. Her mental status and functional status are normal. Question What should you tell the patient?

"You have sleep changes that are normal with aging."

A 50-year-old man is a new patient to the practice. You know his wife fairly well, but the patient has not come in for visits as he is an owner of a busy restaurant and rarely leaves the restaurant during the week. He comes in today for acute bronchitis. After completing the exam and writing prescriptions for antibiotics and a beta-agonist inhaler, you sit down to assess where he is in terms of smoking cessation. It is obvious to you that he has been a smoker for most of his life from the presence of the cigarettes in his shirt pocket, the smell of tobacco on his clothes, and the staining of his teeth. The patient surprises you by saying that he has been thinking about stopping smoking for the past 3 months and even borrowed a nicotine patch from his friend to try out; he had not got around to doing that yet. When asked why he wanted to stop, he said that he loved to smoke, but he hated the amount of money he spent on cigarettes each week. He figured that he could save between $1,000-1,500 each year if he stopped smoking. At this point, you congratulate the patient for his courage in wanting to try, showing understanding at how difficult it is to break a habit that he has been engaged in since he was 10 years of age. You recommend that he use pharmacotherapy to help him in his smoking cessation efforts, as he had intuitively wanted to do. Question What should you tell the patient to expect?

A quit rate of better than 30% with combination therapy

A 42-year-old man has been divorced 3 months. He has presented with complaints of an inability to maintain long-term relationships. He also states that he has had trouble sleeping and that he recently was fired from his job. He appears to be very self-confident. Question What is the most likely diagnosis?

Adjustment disorder Psychosocial stressors, such as a divorce, can trigger emotional or behavioral symptoms (manifested by sleeping problems). Given that the patient's divorce was only 3 months ago, his case meets the onset criteria for Adjustment Disorder. Impairment in occupational (getting fired) or social functioning fulfills another criterion for this disorder. In a case where the stressor can have long-term consequences (such as in a divorce, where there may be financial and emotional difficulties in adjusting), this diagnosis may continue to be used beyond the 6-month adjustment criterion outlined in the DSM5. Patients diagnosed with Narcissistic Personality Disorder tend to be grandiose in their self-appraisals. The patient's presentation of self-confidence would not be interpreted as an over-inflated sense of self.

A woman in her mid 30's is seen in your office for the first time. She was court-ordered into treatment due to a recent arrest. The patient says that about a month ago she was out getting high one night and "things got out of control". According to the police report, the patient was found high in a tree, naked. She was disoriented, hyperalert, hostile, and aggressive towards the police officers. She was extremely talkative and she complained of severe abdominal pain. The medical examination that followed revealed malnutrition, hypertension, pupillary dilation, tremulousness, tachycardia, repetitious compulsive behavior, and hepatitis C. The patient tells you that she remembers visual and auditory hallucinations, particularly voices that criticized her behavior, and that following the episode she slept for 18-20 hours per day for about 3 days. Significant depression has ensued since. The patient admits that she used an illicit drug and that it led to this episode; however, out of shame, she is initially unwilling to tell you what drug she used. Question What is the most likely diagnosis?

Amphetamine abuse Amphetamine and Cocaine abuse present similarly but amphetamine abuse is usually followed by a period of depression that can last several weeks.

A 25-year-old man presents with euphoria and has clinical symptoms that include tachycardia and nausea. He admits to having used methylphenidate earlier in the day. He also smokes marijuana occasionally. How would you categorize his intoxication?

Amphetamine-related disorder

An 18-year-old boy frequently stays out late at night with his friends, despite his parents forbidding it. He ran away from home several times when he was younger, and he is frequently truant from school. His grades are poor, and he has been held back a grade in school and placed in special education classes. The school psychologist gave him a diagnosis of conduct disorder when he was 13. When he attends school now, he threatens and intimidates students and teachers, and he seems to enjoy getting into fights. He has attacked one student with a brick, and he boasts that he carries a knife and has a gun hidden on school grounds. The boy has never expressed feelings of remorse for his actions. A good talker, he often engages in conning his teachers and enjoys the attention it seems to get from the other kids. He has been shoplifting for the past several years. Question What is the most likely diagnosis?

Antisocial personality disorder Antisocial Personality Disorder is not diagnosed before age 18 and requires that there is evidence of Conduct Disorder before age 15.

A 27-year old accountant who lives with his mother and has an intense concern that he will be criticized or disapproved of. He steers clear of after-work activities with his fellow workers. He has never had sex, but he is not homosexual and finds women attractive. Needing some assurance that he will be liked, he has problems in social situations and finds it difficult to date because he views himself as inept or inferior to others. Never having had a chance to engage in sports activities as a youngster, he does not want to try his hand at tennis because he may be embarrassed by his poor showing. Question What is the most likely diagnosis?

Avoidant personality disorder

An 18-year-old woman presents with weight loss. Her friend is worried about her and provides most of the patient's history. The friend notes that the patient has become very thin over the past several months and that her attitude is "listless." The patient's mother died 5 years ago, and the patient admits not feeling comfortable with her father's new wife. She is starting college and admits that she never feels hungry. She is not interested in food; she avoids social situations that involve meals and frequently worries about her appearance. She has lost 25 lb in 7 months. She has not had a menstrual period for 4 months. Her BMI is 17.6, and her vital signs are normal. Physical exam is remarkable for a thin-appearing female with lanugo hair on her arms. Laboratory evaluation is remarkable for a potassium of 3.3 mmol/L (normal 3.4-4.9) and a low total protein and serum albumin. Weight106 lbHeight65"Body mass index17.6Pulse98Blood pressure108/72 mm HgTemperature97.1°F/36.1°C Lab results: TestResultNormal rangeUnitsComplete blood count (CBC)WBC6.13.6-9.0K/μLRBC4.784.18-5.22M/μLHemoglobin12.612.9-15.5g/dLHematocrit34.534.6-50.1%MCV7980.0-100.0fLMCH32.227.0-34.0pgMCHC34.130.0-37.0g/dLRDW11.711.0-17.0%Platelets378140-440K/μLMPV9.96.5-12.0fLWBC differentialNormal Complete Metabolic Panel (CMP)Sodium134134-144mmol/LPotassium3.33.4-4.9mmol/LChloride100100-109mmol/LHCO32620-31mmol/LGlucose7270-99mg/dLBun187-18mg/dLCreatinine0.90.6-1.2mg/dLCalcium9.98.8-10.5mg/dLAlbumin3.43.5-5.0g/dLTotal Protein6.16.4-8.2g/dLAST (SGOT)2115-37U/LALT (SGPT)175-43U/LAlk Phosphatase5550-136U/LTotal Bilirubin0.80.1-1.2mg/dLeGFR>60>60mL/min/1.73 m2Human chorionic gonadotropin (hCG)Negative Thyroid-stimulating hormone (TSH)2.1410.500-4.700uIU/mL Question What is the best intervention?

Begin multidisciplinary treatment approach.

A 35-year old woman is very concerned that she will be abandoned by her boyfriend. She has always been very intense in her relationships; she frequently alternates between idealization and devaluation, is often highly impulsive, and engages in reckless driving and casual sex. When left alone, her thoughts turn to suicide, and she has made several gestures in the past. Her mood ranges from intense dysphoria to anger, and she sometimes suffers from feelings of dissociation. Question What is the most likely diagnosis?

Borderline

A 20-year-old woman presents for therapy after being released from the emergency department. She was brought to the emergency department by her significant other because she was making superficial cuts on her wrists in response to a fight they had; the significant other walked out during the fight. She is tearful and vehemently angry at her partner. However, she is also comforted to know that the relationship is not over. The partner relates that this has happened several times before, and he believes that if the relationship ever did end, that the girl would "kill herself once and for all". You suspect that your patient is suffering from a personality disorder. Question What is the most likely diagnosis?

Borderline Borderline personality disorder is most often associated with a pattern of unstable interpersonal relationships.

A 19-year-old female student is referred for psychiatric assessment after an attempted suicide 2 days ago. On questioning, the patient informs that the precipitant to her attempted suicide was a recent breakup with her boyfriend. Her history is significant for multiple suicide attempts since the age of 12. On further questioning, the patient informs that she has a chronic feeling of emptiness and sometimes feels disconnected from reality. She also gets into fights with her boyfriends, as she has difficulty controlling her anger. The patient admits that she has had highly problematic relationships with men and that she falls quickly in and out of love. Examination reveals several cuts on the wrists and arms. Question What personality disorder does this patient have?

Borderline This psychiatric condition is characterized by emotional turmoil, affective instability, chronic suicidal behavior (suicide ideation and attempts), and instability of interpersonal relationships.

A 25-year-old man states that he feels sad and lonely all the time. He rarely speaks with his parents, because he feels that they do not understand him or care about him as much as they do his siblings. Although he went to college for a while, he never seemed able to settle on a major. He has a poor work history, and he frequently changes careers. Further, he complains that all of his romantic relationships seem to fail. He says his relationships always start out great, but after a few weeks, things always change. His girlfriends always pull away from him, and they stop respecting his needs. He often tells his girlfriends that if they leave him, he will kill himself. These threats are frequently followed by intense displays of anger. He has been hospitalized 1 time for overdosing on medication and 2 times for superficial cuts to his arm. Question What is the most likely diagnosis?

Borderline personality disorder

A 27-year-old man underwent pre-employment urine drug testing of hospital employees after recently being hired for a full-time position. Although the patient admits to using both legal and recreational drugs in the past, he claims he has not used anything for at least 2 weeks. The urine test returns positive. Question What substance is the man most likely abusing?

Cannabinoids The only drug from the list that is detectable in urine after 2 weeks is marijuana (cannabinoids). Approximate duration of detectability in urine of chronic heavy use of marijuana is 36 days. Single use is detectable for 1-2 days and moderate (daily) use is about 10 days. Nicotine is detectable in urine for about 12 hours. Amphetamine is detectable in urine for 2-3 days. Cocaine is detectable in the urine for about 6-8 hours. The approximate duration of detectability in urine of opiates and their metabolites does not exceed 3-4 days.

A 23-year-old man presents for the first time in a mental health clinic where you consult. He walks in and slumps into a chair opposite yours and he pulls out a candy bar. "I know it's only been an hour since lunch, but I'm really hungry. Do you mind?" Quick observation reveals no remarkable difficulties in gait, coordination, attention, or speech. The patient reports that a judge has sent him in for an evaluation after being arrested for possession of a small amount of marijuana. He is on welfare and otherwise begs to make money. He has not worked in years because he finds that the combination of welfare and begging meets his needs most easily. When he is not begging, the patient admits that he "takes a toke every now and then". He denies using other drugs or alcohol, although he has tried most of them. He claims that none, other than marijuana, has done him much good. About 20 minutes into the session, the patient stands up, rubs his bloodshot eyes, and thanks you for listening. When you tell him that only 20 minutes have passed, the patient—looking startled—says "Gee, it seems like it's been at least an hour!" and he slouches back into the chair. Question What is the most likely diagnosis?

Cannabis intoxication Diagnostic Criterion B for cannabis intoxication states the essential feature of clinically significant problematic behavioral or psychological changes that develop during, or shortly after, cannabis use. According to the DSM-5, this patient's time distortion fulfills the requirement for a maladaptive behavior. Other symptoms of cannabis intoxication include red eyes, increase in appetite, dry mouth, and rapid heart rate, each of which is neither the result of a general medical condition nor better explained by another mental disorder. This patient exhibited red eyes, increased appetite, and a lack of motivation, in addition to time distortion. Cannabis abuse is characterized by regular daily intake of large amounts of marijuana, regular heavy use limited to weekends, or binge periods of heavy use interspersed with long periods of abstinence, despite negative consequences when the substance is used. Marijuana is a central nervous system depressant.

A 6-year-old boy frequently gets out of bed at night and walks around the house without appearing to awaken. What step would be best to undertake first?

Cautionary safety measures As children often outgrow sleep walking and this child is only 6, neither counseling nor medication is necessary at this point in time.

A 58-year-old man is admitted for seizure evaluation and management. After he returned from work in the evening, the man collapsed to the floor and had writhing movements in all of his limbs and frothing at the mouth; the movements and frothing lasted for 4 - 5 minutes. His wife noted that he had fecal and urinary incontinence. The patient informed you that he felt groggy after regaining consciousness, but it soon resolved. He denied chest pain or headache. The patient confirms that he quit smoking 7 years back, but he smoked for 20 years prior to that. He also states that he drinks 4 pints of beer and 2 shots of 'vodka chasers' every evening when he returns from work. His wife discretely mentions that since his friend's death 2 months back, he has been drinking excessively; however, he has refrained for 2 days. His blood pressure is 140/94 mm Hg and his pulse is 86/minute; there are no changes in these parameters while lying or standing. His temperature is 97.4°F, and his respirations are 16/minute. On examination, the patient is alert and orientated, has a ruddy appearance, Dupuytren's contracture of his left hand, and no hepatomegaly. His neurological exam is normal and he has no tremor. His latest lab work is as follows: Hemoglobin15.9g/dLHematocrit46.2%White Blood Cell count9.3x109/LPlatelets207x109/LSodium137mmol/LPotassium4.0mmol/LChloride96mmol/LBicarbonate23mmol/LBlood Urea Nitrogen11mmol/LCreatinine0.9mmol/LALT149 IU/LAST235 IU/L Question What agent would best reduce the likelihood of further seizures in this case?

Chlordiazepoxide and other benzodiazepines have documented efficacy in reducing seizure episodes, and they are recommended as suitable agents for alcohol withdrawal syndrome.

The best diagnosis would be female orgasmic disorder, situational. The woman has previous experiences of orgasms with her husband and is still able to achieve an orgasm by masturbating. Additionally, this would rule out a diagnosis of female orgasmic disorder, lifelong. Anorgasmia is characterized by a failure to achieve orgasm; because the woman is still able to achieve orgasms, this is not the best diagnosis.

Cocaine-related disorder

An active 10-year-old boy stays out late at night and has a history of running away from home and skipping school. When he attends school, he bullies other children and seems to enjoy getting into fights. He previously attacked another student with a brick and destroyed library books for fun. He is imaginative and often has a good cover story for where he has been when something has been stolen or is broken. A favorite weekend activity for the past 6 months has been shoplifting. Question What is the most likely diagnosis?

Conduct disorder

A 60-year-old woman presents with short-term memory loss, an inability to care for herself, and an inability to handle finances. According to her neighbor, she recently left something on a burning stove and almost burned her house down. There is no family with her. Question What is the next step in patient management?

Contact adult protective services

A 36-year-old man with a history of diabetes and obesity presents with weakness and flu-like symptoms. His girlfriend reports that he had taken several caffeine pills the day before, but he denies a suicide attempt. During evaluation at the hospital, he experiences vomiting and seizures. A laboratory workup is within normal limits. He is admitted to the hospital and improves over the course of his 3-day admission. He is subsequently released with no lingering effects. Question What is the most common treatment for this patient after hospital discharge?

Decrease or stop caffeine intake

A 53-year-old man presents with a 2-hour history of unusual behavior. The patient is accompanied by a family friend; the friend tells you that the man has been acting "spaced out and strange". The history reveals that this man has no personal or family history of mental disorders; his symptoms have occurred on a daily basis over the past several weeks. They occurred for the first time "out of the blue", and the symptoms come and go in the course of a typical day. On examination, the man appears to be in no apparent distress, but his speech is obviously rambling; at times, it is incoherent. His attention wanders, and he periodically nods off during the short interview. The man seems to be responding to visual stimuli. The agitated patient reports that he has had little sleep over the past several weeks, and he attributes his condition to this fact. There is no indication that substance use is involved, and the only significant laboratory finding is that of a thiamin deficiency. Question What is the most likely diagnosis?

Delirium Delirium is more sudden onset than dementia Amnestic disorders are also characterized by an impairment of memory, but in contrast to delirium and dementia, amnestic disorders are not characterized by global deficits in intellectual and cognitive functioning.

A 75-year-old woman is in good general health and is known to your practice. Over time, you have begun to notice short- and long-term memory impairment. When questioned, she angrily denies having any problems, but her children admit that they have been concerned that she does not seem to be "as sharp" as she used to be. She has no known past medical history and her physical examination and lab work are within normal limits Question What is the most likely diagnosis?

Dementia

A 55-year-old man has had emotional changes consisting of sexual disinhibition and emotional apathy. He later has difficulty with expressing himself and doing his usual routine. The magnetic resonance imaging (MRI) reveals atrophy of the frontal and temporal lobes, along with swollen neurons in the temporal and frontal areas of the brain, and intraneuronal argentophilic inclusion bodies in these same areas. Question What type of dementia is this patient exhibiting?

Dementia due to Pick's disease (Frontotemporal Dementia)

A 70-year-old woman presents with a 1-year history of short-term memory loss. She did not seek medical attention earlier because she believed the memory loss to be age-related. Her husband has also noticed that she is unable to perform her regular daily activities and is often misplacing things. Recently, she was unable to get back home after her evening walk; neighbors brought her home. The husband also feels she has been acting strangely, and she is suspicious of her own son. Her past medical history is significant for hypertension, and she has been on medication for the past 3 years. On clinical examination, she appears conscious and alert; PR is 70/min and BP is 120/74 mm Hg. She is afebrile. Question What is the provisional diagnosis?

Dementia of Alzheimer's Disease Vascular dementia has the same general symptoms and signs as Alzheimer's disease, except the onset is usually more sudden and acute. In addition, it is picked up easily by clinical or laboratory evidence of a vascular cause, a history of cerebrovascular disease, or multiple infarctions.

A 35-year-old man presents for counseling because his relationship of 5 years just broke off; his partner ended the relationship. He says that he is unsure what to do because he has not been without a significant other for more than a week or so since high school. You determine that the man is not suicidal or homicidal; however, from his responses to your questions you get the sense that, unless he enters into a new relationship, this man's sense of loss will persist long after a normal period of adjustment. Question What is the most likely diagnosis?

Dependent

A 25-year-old man comes to see you after breaking up with his girlfriend. He is in a panic. He tells you that he has never been on his own and wants to know what you think he should do. He says he does not understand why his girlfriend broke up with him. He always did everything that she asked of him and never disagreed with her. "All I wanted to do was make her happy." You asked him if his ex-girlfriend gave him a reason for the breakup. She told him that she was tired of making all of the decisions in the relationship that they should make together: where to go for dinner, which apartment they should choose, etc. The client tells you that he must find a new girlfriend quick or his whole life will fall apart. Question What is the most likely DSM-IV Axis II diagnosis for this patient?

Dependent personality disorder

A 28-year-old man lives with his parents and works in his family's business. Although he handles his work responsibilities without difficulty, he lets his mother handle his money, pay his bills, and purchase his clothing for him because he gets anxious about making the "wrong" decisions. He rarely goes out alone, but when he does, he lets others make plans. Question What is the most likely diagnosis?

Dependent personality disorder

A 73-year-old white male visits his family physician because of problems sleeping. He goes to sleep each night at 9:00 p.m. and promptly falls asleep. He wakes up about 5 times during the night. In the morning, he feels tired, and he finds himself sleeping during the day. Tape recordings during 2 successive nights reveal no unusual noises or snoring. He denies alcohol and caffeine. His breathing has been "a little shorter" recently. He has a past medical history of congestive heart failure (CHF), atrial fibrillation, chronic obstructive pulmonary disease (COPD), and benign prostatic hypertrophy (BPH). His medication list includes digoxin, theophylline, and furosemide. He says he takes his medications faithfully as directed: digoxin, furosemide, and theophylline before bedtime. His physical examination shows a normal ear, nose, and throat exam. He has normal heart sounds. His breath sounds confirm rales heard 1/3 up both lungs. He has pitting edema of the ankles. He has a large soft prostate. His post-void residual volume is 400 milliliters (normal is 200 milliliters). His height and weight are appropriate. Question What is the next step in the patient's management that will immediately improve his quality of sleep?

Discontinue the theophylline has "caffeine-like" quality

A 70-year-old woman presents to be evaluated for bizarre behavior. Her daughter arrives with her and speaks with you alone; she describes her mother's behavior as consisting of mood swings, lavish trips, spending foolishly, staying up at night, and being hyper. According to her daughter, her mother has been diagnosed with bipolar disorder in the past. Past medical history is significant for chronic kidney disease stage III, obesity, diabetes mellitus, and hypertension. Question What drug might you consider?

Divalproex (Depakote) is a preferred or suggested drug as a mood stabilizer; it has also been used to treat aggressive behavior in patients with Alzheimer's disease. It has been shown to be more favorable than using lithium, especially in nursing homes, because lithium can cause excessive weight gain. Lithium should not be used in patients with kidney disease. Divalproex (Depakote) should not be used in elderly patients who already have poor liver functioning.

An 80-year-old woman is diagnosed with dementia of the Alzheimer's type as a result of comprehensive testing. You believe she is in the very early stages of the disease and you want to try a medication to possibly slow disease progression. Question What drug would you be most likely to try first in this patient?

Donezepil (Aricept)

A 25-year-old Caucasian woman comes into the clinic for her appointment. She is complaining of not having a period for the last 6 months. She states that she has not had the money to see a doctor until now. She started her menstrual cycles at age 12, and they have been mostly irregular. Her flow lasted 5 days without significant cramping. She is a G 0 P 0 AB 0. Her vital signs are within normal limits. She is 5'10", 100 lb. A urine pregnancy test is negative. She states that she is just too fat. When asked how much she exercises, she replies that she runs 10 miles every day. Question What other information would be the first that you would want to know?

Eating habits

A 55-year-old woman has advanced breast cancer and a poor prognosis. She has a teenage son. You have already talked to this patient about her disease and its progress, but she is in denial and doesn't want to discuss her cancer with anybody, including her son. The hospice counselor has spoken to her and her son separately. Question What might opening the lines of communication between this patient and her son allow for?

Feeling safe to ask questions

A woman reports being unable to experience an orgasm with her husband (with whom she has previously experienced orgasms), but reports being able to achieve orgasm through masturbation. Question What is the most likely diagnosis?

Female Orgasmic Disorder, Situational

A 23-year-old woman presents for a routine check-up. She has no current complaints but admits to using laxatives and diuretics on an average of once a week after she consumes an enormous amount of food. She denies self-induced vomiting and does not have physical signs that would raise concern. She also denies alcohol and drug use. Her physical examination is within normal limits, including her weight and vital signs. Question In addition to psychotherapy, what pharmacologic therapy should be prescribed as a first-line agent in order to treat her disorder?

Fluoxetine (first-line for bulimia)

A 26-year-old man is brought to the ER by his friends; they state that they were at a party where the patient drank approximately 6 beers and ingested an unknown substance. He is unresponsive and hypothermic (35.2°C/95.36°F), respirations are labored, and Glasgow Coma Score equals 3. Toxicology screen is negative, and blood alcohol concentration is 50 mg/dL. He is rapid sequenced and intubated. Naloxone is administered without improvement of symptoms. An hour later he awakens suddenly and self-extubates. Question What is most likely the unknown substance ingested?

Gamma-hydroxybutyric acid (GHB) structurally similar to the inhibitory neurotransmitter gamma-aminobutyric acid (GABA) and stimulates the release of dopamine from the basal ganglia. Unfortunately, it is becoming more common as a substance of recreational drug overdose. Classic presentation includes severely depressed level of consciousness with sudden awaking within several hours, hypothermia, and respiratory difficulty. The other choices would be excluded by a complete toxicology screening.

A mother seeks medical attention for her 7-year-old son. For the last 8 months, he has not acted like his three older brothers. The mother indicates that her son has said on repeated occasions he wants to get rid of his male genitalia and he would prefer to be a girl. She has found him wearing his sister's clothing on numerous occasions. More history shows that he prefers to play with dolls and only spends time with female friends. Teachers relate that he turns down invitations from the boys in the class to join in sports activities. His male classmates are now teasing and embarrassing him in class, and it has begun to affect his schoolwork. Question What is the child's most likely diagnosis?

Gender dysphoria

A 25-year-old woman presents with inability to achieve intercourse with her partner since their relationship began 6 months prior. She states that she cannot even insert a tampon due to the pain. Further history reveals that a year ago, her partner at that time forcefully made her have intercourse on several occasions. What is the next step in the workup of this patient to confirm the suspected diagnosis?

Gynecologic examination (Vaginismus)

A 60-year-old man with a history of HIV, poorly controlled hypertension, and diabetes presents to your office with forgetfulness, slowness, poor concentration, imbalance, ataxia, and saccadic eye movements. CT scan of the brain shows multifocal destruction of the white matter and subcortical structures Question What is the most likely diagnosis?

HIV-associated dementia

A 35-year-old former IV drug user presents with drastic personality changes after her diagnosis with HIV disease 5 years ago; although she has felt physically well during the period, she says that she frequently forgets things that are a normal part of her daily routine. She is tearful and easily distracted; although others have noticed these problems, she angrily denies that anything is wrong when they express concern. A recent neurological examination and complete physical are essentially normal. Question What is the most likely diagnosis?

HIV-related psychiatric disorder

You are called to visit a 70-year-old man in a nursing home with the diagnosis of Alzheimer's dementia, heart failure, diabetes, and a urinary tract infection. He needs to be seen for his fever and infected ulcers. Since last year, he has had stage 3-4 multiple decubitus ulcers in his ankle and his lower back. 8 months ago, he was hospitalized twice for his ulcers. During the visit, you realize that the patient cannot walk without personal assistance and he does not talk more than six intelligent words in the course of your interview. His caregiver tells you that he is unable to dress or bathe without assistance. Over the past weeks, occasionally he has had incontinence of urine and stool. He has 2+ edema in his lower extremities, which decreases with administration of diuretics. His blood pressure is 110/68 mm Hg, his pulse rate is 80/min., and he has rales in the base of both his lungs. He also has had unintentional, progressive weight loss of greater than 10% over prior 6 months. His serum albumin is 2 gm/dl. You administer appropriate antibiotic and wound care. You estimate that his life expectancy is less than 6 months. Question What is your suggestion for this patient's health maintenance?

Hospice care with the diagnosis of end-stage dementia

A 22-year-old female college student presents because she does not eat properly and has missed several menstrual cycles. Her sorority sisters are certain she is not pregnant because she rarely, if ever, leaves their sorority house except to attend classes; she has not dated in more than 6 months. On examination, she is underweight. She walks unaided and her speech is clear and distinct. She has adequate vision, normal-appearing facial expressions, and adequate hearing. On her college entrance physical examination, her height was 5'7" and her weight was 130 lb. Her weight 1 year later is now 103 lb. Question What area of her nervous system is involved with her disorder?

Hypothalamus (anorexia nervosa)

A 16-year-old boy was referred to you for an evaluation due to assaultive behavior. He has been suspended several times for assaulting other students during the lunch break. The school is threatening to expel him if this behavior doesn't stop. During the last incident, the boy broke another student's nose and he was charged with assault. During the interview, the youth is very polite and cooperative. He states he doesn't know what happens. He says that someone will say something that he disagrees with and all of a sudden he is filled with this overwhelming anxiety. The next thing he knows he has hit the other individual. He states he feels bad about what he has done. He also tells you that he is very upset because he doesn't really have any friends and that most of the other students are afraid of him. He says he doesn't plan to hurt other people and that he doesn't necessarily have any negative feelings toward them, he just can't help himself. The client denies ever having any homicidal or suicidal ideation, and he says that for the most part he is very happy. He denies ever feeling like he has too much energy and says he sleeps and eats just fine. He does not have a family history of mental illness and denies any abuse. He is also negative for hallucinations or delusions. Question What is the most likely DSM-5 diagnosis for this client?

Intermittent Explosive Disorder In this case, the youth experiences several discrete episodes of failure to resist aggressive impulses that result in serious assaultive acts or destruction that are grossly out of proportion to any precipitating psychosocial stressors. Further, the youth has expressed remorse for his behavior. These symptoms are consistent with a diagnosis of intermittent explosive disorder. Intermittent explosive disorder can be differentiated from oppositional defiant disorder, conduct disorder, and antisocial personality disorder by the propelling force behind the assaultive behavior. Those with intermittent explosive disorder do not assault people out of revenge or enjoyment in hurting others. They do not plan these acts, and they often feel remorse for what they have done.

A 58-year-old woman is brought to the outpatient clinic by her son. He states that her muscles seem stiff and she cannot walk properly. He also says she has a tremor when standing and has fainted and fallen several times recently. He also noted that she cannot concentrate or remember what he tells her. Last month she forgot to pay her bills. She also looks confused most of the time. CT head was done and it reports no abnormalities. Question What is the most likely diagnosis?

Lewy body dementia Alzheimer's Disease differs from Lewy Body Dementia in that the person's memory and ability to think gradually declines, and there isn't usually a daily variance. There is not usually physical deterioration or gait changes unless there is a comorbid disease until Alzheimer's has significantly progressed. Changes in facial expression don't develop until the later stages.

A parent brings her 16-year-old daughter to the emergency room, after coming home from work early and discovering her daughter "on drugs." While her mother is in the examination room, the patient denies taking any drugs, but admits that she did take "something my friend gave me," when examined alone. She reports that she has taken this substance every few weeks for at least the past year, saying she just does it for "fun" with her friends. She says that today she took the substance by mouth approximately 2 hours prior to presentation. She describes seeing hallucinations of geometric shapes and the appearance of objects changing shapes before her eyes. She also describes "seeing" music becoming the colors red and blue shortly after taking the illicit substance. Physical exam reveals dilated pupils and tachycardia. Question What is the most likely drug involved in this patient's intoxication?

Mescaline (hallucinogen)

A 29-year-old man is found unconscious by his roommate. He has a history of manic-depression and substance abuse. His roommate states that he recently broke up with his girlfriend and was laid off from his job. When the paramedics arrive, the patient is nonresponsive and nearly apneic. His pulse is palpable and regular at a rate of approximately 80 beats per minutes. His pupils are pinpoint but equal. Question In addition to immediate cardiopulmonary resuscitation, treatment with what drug therapy is most likely to be lifesaving?

Naloxone The patient's apnea and pinpoint pupils are consistent with an acute opioid overdose

A 58-year-old woman presents for a follow-up accompanied by her adult daughter. A few months ago, the patient had gone to the emergency department via ambulance after the daughter discovered her mother confused and shaking at home. The patient underwent several days of inpatient treatment for alcohol withdrawal. Initial symptoms included hallucinations, tremor, nausea, anxiety, insomnia, and a seizure. The patient denies current symptoms and admits increased tolerance to greater amounts of alcohol and loss of control with a frequent need for the substance. Her work and home relationships suffer due to her condition. The patient has no other medical conditions, is menopausal, has had no surgeries, takes no medications, and has no allergies. The patient readily admits alcohol abuse and dependence with a desire to prevent relapse. She has not used any other substances, and has been abstinent of all substance use since hospital discharge. She is currently in individual and group programs to assist her in relapse prevention. She would like pharmacological help to maintain her sobriety. Question What medication is most appropriate in relapse prevention for this patient?

Naltrexone (ReVia/Vivitrol) The acute episode described by the patient and her daughter is consistent with delirium tremens, a serious and possibly fatal condition resulting from alcohol withdrawal. Symptoms include seizures, tremor, nausea, anxiety, visual and auditory hallucinations, confusion, and diminished orientation. Delirium tremens can progress to coma and death if not recognized and treated. Alcohol dependence is common, and many patients diminish and hide their alcohol consumption. Acute intervention includes supportive treatment of symptoms and benzodiazepines. In order to maintain abstinence, psychosocial support is vital. In addition, two medications, naltrexone (ReVia/Vivitrol) and acamprosate (Campral), are approved for maintenance treatment of alcohol dependency. Diazepam (Valium) is a benzodiazepine often indicated for acute alcohol withdrawal; it is not appropriate for use in preventing relapse in patients with alcoholism. As a controlled substance with abuse potential, it would be risky to prescribe diazepam to this patient. Methadone (Methadose/Dolophine) is a long-acting opioid used for pain treatment, acute detox, and maintenance treatment of opioid addiction. Acute intoxication symptoms may include sedation, euphoria, pinpoint pupils, unconsciousness, and withdrawal symptoms (e.g., nausea, vomiting, dilated pupils, abdominal cramps, and insomnia). Naloxone (Narcan) is an opioid antagonist used for opioid overdose and reversal. Varenicline (Chantix) affects nicotinic acetylcholine receptors and is approved for smoking cessation.

A 35-year-old woman has a history of difficulty in social relationships. She is highly demanding and hyperfocused on her appearance. When she wants a haircut, she refuses to wait for an appointment; she demands immediate service regardless if others were waiting ahead of her. She frequently ignores her boyfriend at social functions, and even though he goes out of his way to try to please her, she flirts with other men. As a result of these behaviors, she has few close friends. Question What is the most likely diagnosis?

Narcissistic personality disorder

A 20-year-old woman, who works as a receptionist in a wholesale dress house, feels that she has not yet been discovered by the fashion world. She has done some showroom modeling for customers, and she is always daydreaming that she will be swept off her feet by the next wealthy man who comes through the door. When eating in restaurants, she always takes the opportunity to complain about the clumsy waiters, the inept service, or the poor quality of the food. She engages in a rich fantasy life in which she is adored and loved by crowds of onlookers; in her fantasies, she gets even with all the people who she perceives have been mean to her. Question What is the most likely diagnosis?

Narcissistic personality disorder The woman is exhibiting the usual symptoms for a diagnosis of narcissistic personality disorder: a grandiose sense of self-importance; preoccupation with fantasies of unlimited success; a belief of being special; a sense of entitlement; a lack of empathy; and an arrogant attitude toward others. Borderline personality disorder (BPD) is a complex severe mental illness that is characterized by poor interpersonal relationships, mood instability and unstable self-image and behavior. Personality disorders represent "an enduring pattern of inner experience and behavior that deviates markedly from the expectations of the individual's culture" per the Diagnostic and Statistical Manual on Mental Disorders, Fifth Edition (DSM-5). Dependent personality disorder is a psychiatric condition marked by an overreliance on other people to meet one's emotional and physical needs. Histrionic personality disorder is characterized by constant attention-seeking, emotional overreaction, and suggestibility. A person with this condition tends to over-dramatize situations, which may impair relationships and lead to depression.

The onset of rapid eye movement sleep within five minutes of sleep onset is pathognomonic for

Narcolepsy

A 19-year-old man presents for evaluation of excessive daytime sleepiness. Despite getting a full night's sleep, his coworkers have repeatedly noticed him suddenly dozing off at his desk, during meetings, and sometimes mid-conversation. Upon further questioning, the patient admits that he occasionally experiences nighttime episodes of feeling unable to speak or move. The remainder of the history and physical exam are unremarkable. Aside from the occasional ibuprofen for shoulder pain, he does not take any medications. Question The patient's symptoms are most consistent with what disorder?

Narcolepsy four major symptoms: excessive daytime sleepiness cataplexy sleep-related hallucinations sleep paralysis

A patient who has been on a course of neuroleptic medication for the past few weeks has begun exhibiting a resting tremor affecting his limbs, head, and jaw; his mouth has shown a classic "rabbit syndrome". In addition, there is muscular rigidity, akinesia, a feeling of listlessness, a staring facial expression, small shuffling steps, and some drooling. When he writes, the patient now uses small handwriting, and there is inhibited blinking in response to glabellae tapping. Question What is the most likely diagnosis?

Neuroleptic-induced Parkinsonism

An 8-year old boy has become enuretic at night, once a week on average; this has happened for the past 2 months. He is on no diuretics. There is no general medical condition, such as diabetes, spina bifida, or seizure disorder. He appears to be in good health. No first-degree relatives have similar problems. On exam, no lab values outside normal range are found, and there is no apparent reason for his problem. Question What is the most likely diagnosis?

No diagnosis due to lack of diagnostic criteria Separation anxiety disorder consists of persistent and excessive anxiety beyond that expected for the child's developmental level related to separation or impending separation from the attachment figure (primary caretaker, close family member). The Diagnostic and Statistical Manual of Mental Disorders, Fifth Edition (DSM-5), classifies enuresis under the heading of elimination disorders. DSM-5 criteria for enuresis are as follows: Repeated voiding of urine into bed or clothes, whether involuntary or intentional The behavior either (a) occurs at least twice a week for at least 3 consecutive months or (b) results in clinically significant distress or social, functional, or academic impairment The behavior occurs in a child who is at least 5 years old (or has reached the equivalent developmental level) The behavior cannot be attributed to the physiologic effects of a substance or other medical condition. According to the Diagnostic and Statistical Manual of Mental Disorders, Fifth Edition, (DSM-5) symptoms of Reactive Attachment Disorder (RAD) must be evident before five years of age and the child must have a developmental age of at least nine months. The child often shows consistent emotionally withdrawn behavior towards caregivers; specifically, he/she minimally responds or seeks comfort when he/she is emotionally distressed. The second criterion includes a limited positive affect, episodes of unexplained irritability, sadness or emotional turmoil during non-threatening interactions with caregivers and minimal social or emotional responsiveness to others. The third criterion is characterized by a pattern of insufficient care such as neglect, physical or sexual abuse, or repeated changes between primary caregivers. The specific DSM-5 diagnostic criteria for Disinhibited social engagement disorder (DSED) are as follows: A pattern of behavior in which a child actively approaches and interacts with unfamiliar adults in an impulsive, incautious, and overfamiliar way The behaviors described in the first criterion are not limited to impulsivity but also include socially disinhibited behavior A pattern of extremes of insufficient care The care described in the third criterion is presumed to be responsible for the disturbed behavior described in the first criterion The child has a developmental age of at least 9 months The diagnosis is not made when there is a medical condition or a medication which could account for the incontinence.

You have been working with a 13-year-old girl. Her school referred her to you. The referral paperwork stated that she refuses to follow the rules at school and frequently talks back to her teachers. In conversations with her mother, the mother states that she refuses to help out with chores around the house and comes and goes as she pleases. Her mother states the child often starts yelling at her when asked to do simple chores around the house. When asked how long this behavior has been occurring, the mother stated she has been like this for at least one year. She has no reported history of arrests, drug use, or violent behavior. She currently denies any depression or suicidal behavior. During the interview, she is uncooperative and often sarcastic. Question What is the most likely DSM-5 diagnosis for this client?

Oppositional Defiant Disorder Oppositional defiant disorder can be differentiated from conduct disorder and antisocial personality disorder by the severity of the symptoms. Those with oppositional defiant disorder do not display the assaultive and criminal behavior, or violation of age-appropriate societal norms. Further, in order to be diagnosed with antisocial personality disorder, a person has to be at least 18 years of age.

A 75-year-old woman presents with her daughter. She has lived alone since her husband died 5 years ago of lung cancer. Her daughter tells you that, over the last 2 months, her mother has been having difficulty doing household chores. She also seems to be more short-tempered lately. In addition, she forgot her home address 2 times while returning from the supermarket and had to be escorted home by her neighbor. She is suffering from osteoarthritis and takes ibuprofen intermittently. She is using a salbutamol inhaler along with montelukast to control her 20-year history of asthma. Her sister is suffering from some sort of memory problem and now lives with her son. She denies use of alcohol, tobacco, or illicit drugs.General physical exam is normal. Temperature 37°C, heart rate 70 beats per minute, respiratory rate 16/minute, blood pressure 110/70 mm Hg. Neurological exam reveals that the patient is alert and oriented. Her speech has frequent pauses and she has difficulty finding words. She is able to register 3 items, but she is unable to repeat them after 2 minutes. She does not know today's date or her date of birth. She is unable to draw a clock or copy a cube. The patient's mini-mental status examination is 20/30. Her cranial nerves II through XII are intact, and muscle strength and cutaneous sensation are normal. Deep tendon reflexes are normal.Results of lab tests are shown in the table: Hemoglobin13.8g/dlLeukocyte count5,000/mlNeutrophil %60%Platelet count355,000/mlGlucose120mg/dlSodium140mmol/dlPotassium4.7mmol/dlChloride105mmol/dlBicarbonate22mmol/dlTSH3U/mlVitamin B12250pg/ml (normal 180-1,000pg/ml)Folic acid6ng/ml (normal 3.1-12.4ng/ml) Question What is the most common immediate cause of death in persons suffering from the disorder described in this patient?

PNA (Alzheimer's) Although the disease is the underlying cause of death in about 70% of patients with Alzheimer's dementia, pneumonia and dehydration are the most frequent immediate causes of death. Cancer has been found to be a less likely cause of death in patients with dementia compared to the general population.

Which of the following is a constituent of coenzyme A, glucose production from lipids and amino acids, and steroid hormone synthesis? Vitamin B1 (thiamine) Vitamin B2 (riboflavin) Vitamin B3 (niacin) Pantothenic acid Biotin

Pantothenic acid

From the time she was a child, a 24-year-old woman has always had a problem with her peers. She has never had a friend in whom she felt she could confide because she suspects the motives of everyone. When any remark is made as a joke, she takes it personally and has a tendency to carry a grudge. She feels that she is the constant butt of attacks on her reputation and this has caused a great deal of difficulty in any on-going relationship she has had with a man. She is always questioning their fidelity without grounds to do so. It has resulted in most of the relationships breaking up rather quickly, and she feels she's been cheated by life. There is no history of psychotic illness, no mood disorder, no familial history of psychosis, or serious medical illness. She is in good physical health for her age. What is this woman's diagnosis?

Paranoid Personality Disorder

A 29-year-old male who is an architect by occupation approaches a psychiatrist regarding sexual dysfunction. He has been married for 5 years and has 1 child, a 3-year-old daughter. He complains of ejaculation prior to complete penetration. He says that this problem leaves his wife unsatisfied, and as a result is causing problems in his married life. The medical, physical, and neuropsychiatric examinations determine that he is medically, physically, and mentally healthy, with no evidence of any thought disorder or mood disorder. Question Which of the following would be the most reasonable therapeutic option?

Paroxetine (SSRI for premature ejaculation)

A 14-year-old boy in your practice comes to the office with his mother to discuss his worsening behavioral problems. He has had repeated episodes of violence against neighborhood pets and children, has been arrested 3 times for stealing and damage to others' property, and has been suspended 4 times from school for misconduct. A school counselor had diagnosed him with oppositional defiant disorder, but you think that conduct disorder is probably a more accurate diagnosis given his repeated acts of aggression. His mother tearfully asks whether a sedating medication may be helpful in controlling his behavior, saying that as a single mother she cannot take anymore of his disobedience. Question What should you tell her?

Patients with conduct disorder often do not improve unless removed from their current environment

A 19-year-old college student is brought to the emergency room by the campus police. He was found naked outside a sorority house screaming obscenities and beating his fists on the ground. He is cooperative in the examination room but appears flushed and anxious, and he is holding his hands over his ears. Vital signs are as follows: Temp 99.8, HR 140, and BP 170/110 mmHg. His neurological exam is notable for horizontal nystagmus and minimal pain response to pressing his nailbed. Question What is the most likely diagnosis?

Phencyclidine intoxication Phencyclidine intoxication is characterized by disinhibition, aggressive behavior, anxiety, panic, rage and impaired judgment. Physiological changes can include hyperthermia, elevated blood pressure, tachycardia, hyperacusis, nystagmus, diminished pain responsiveness, dysarthria, and seizures. Opioid intoxication is characterized by behavioral changes that include euphoria followed by apathy, shortened attention span and memory dysfunction, psychomotor agitation or retardation and poor judgment. Physiological changes that may present include slurred speech, drowsiness, and pupillary constriction. Benzodiazepine intoxication is characterized by disinhibition, mood lability and impaired judgment. Physiological symptoms may include slurred speech, poor coordination, nystagmus, impaired memory, and coma. Cocaine intoxication is characterized by behavioral changes that include euphoria, hypervigilance and paranoia, interpersonal sensitivity, anxiety and poor judgment. There are also possible physiological changes that include pupillary dilation, tachycardia or bradycardia, elevated or lowered blood pressure, psychomotor agitation, cardiac arrhythmias, seizures, and diaphoresis. Inhalant intoxication is characterized by behavioral changes that include apathy, assaultiveness and impaired judgment. Physiological changes include nystagmus, slurred speech, psychomotor retardation, as well as movement abnormalities, such as incoordination, depressed reflexes, ataxic gait, and tremor. Mood presentation can range from euphoria to stupor.

A 17-year-old boy has become assaultive and belligerent. He has been having problems with his peers at work, appears to have diminished responsiveness to pain, and he shows signs of hypertension and affective dulling. There is impairment in his judgment, some dysarthria, and a noticeable horizontal nystagmus. Question What is the most likely diagnosis? Phencyclidine intoxication Substance-induced psychotic disorder Antisocial personality disorder Schizophrenia Brief psychotic disorder

Phencyclidine intoxication The correct answer is phencyclidine intoxication. Some of the hallmarks of a substance-induced problem are the presence of aggressive behavior involving fighting, vertical or horizontal nystagmus, agitation, impaired judgment, and impaired social or occupational functioning. Hallucinations may be present, but they are not always; if hallucinations are present, the patient usually knows that they are substance-induced. In this case, the patient did not report a feeling of paranoia. There is no basis for a diagnosis of antisocial personality disorder, and there is no evidence of psychosis. A careful history should include an inquiry regarding any substances used. Drug-induced psychosis, also known as substance-induced psychotic disorder, is simply any psychotic episode that is related to the abuse of an intoxicant. This can occur from taking too much of a certain drug, having an adverse reaction after mixing substances, during withdrawal from a drug, or if the individual has underlying mental health issues. Antisocial personality disorder is characterized by a pattern of disregard for and violation of the rights of others. The diagnosis of antisocial personality disorder is not given to individuals under the age of 18 but is given only if there is a history of some symptoms of conduct disorder before age 15. Brief Psychotic Disorder has a sudden onset of at least one psychotic symptom ( delusions, hallucinations, disorganized speech, grossly disorganized or catatonic behavior). The psychotic symptoms last less than 1 month and are followed by a full recovery. Schizophrenia is characterized by some combination of hallucinations, delusions, and extremely disordered thinking and behavior that impairs daily functioning, and can be disabling.

An 80-year-old woman lives in a healthcare facility. Up until a month ago, she had been self-sufficient, cheerful, and engaged in daily activities with the other residents. Recently, her demeanor has changed, and she now complains of fatigue and generalized pain. She is also irritable and having sleep problems. The physical exam is unremarkable. Laboratory investigation reveals the following: Ca2+ 13 mg/dL, Hct 31%, K+ 3.5 mEq/L, Cl- 98 mEq/L. Albumin is normal. Question What is the most likely explanation for the patient's symptoms?

Primary hyperparathyroidism

A 16-year-old girl presents with a 4-hour history of loss of consciousness. Her older sister states that the patient has been depressed and was found with an empty bottle of tuberculosis medication. The patient has now begun having generalized tonic-clonic seizures. Examination reveals a developed teenage girl responsive only to painful stimuli; there are intermittent tonic-clonic movements of the extremities. Question Besides the general care of an unconscious patient with seizures, what should be given as a specific antidote?

Pyridoxine Isoniazid (INH) is an antibiotic commonly used for tuberculosis prophylaxis. It binds to pyridoxal-5-phosphate, the active form of pyridoxine, which is a cofactor in GABA synthesis. An overdose of INH can result in decreased GABA levels, causing cerebral excitability and seizures. Seizures in acute INH overdose are frequently refractory to standard anticonvulsants. Pyridoxine (vitamin B6) administered on a gram-for-gram basis with the amount of INH ingested is usually needed for seizure control.

An 80-year-old white male who resides at an adult home has been noted to be wandering the floors at night and is also very restless. It takes effort to get the patient to bed and to sleep. A staff physician interviews the patient in the morning. The patient denies any problems of sleep but allows that the night staff is concerned about his confusion at night. The patient has dementia of Alzheimer's type. The patient is not on any medications. His physical examination is normal and he appears orientated to time, place and person. Question The next step of management to help the patient sleep would be to prescribe

Risperidone In the presence of mild Alzheimer's dementia and with a good account of "sundowning", low doses of an antipsychotic (risperidone) given before dark help greatly. A quiet nostalgic evening environment and increased exercise earlier in the day help some patients. Major tranquilizers may help in delirium or dementia-associated disorders.

A 72-year-old woman with mild Alzheimer's dementia resides in a nursing home. In the daytime, she is reasonably high-functioning and oriented to person, place, and time. She is able to carry out activities of daily living independently. In the evening, however, she has difficulty sleeping; on multiple occasions, she has been found wandering the hallways appearing confused and disoriented. When confronted by staff on these occasions, she typically becomes agitated and at times aggressive. A geriatrician interviews the patient in the morning, and the patient admits, "I just cannot fall asleep", but she has little memory of her nighttime confusion and aggression. Her mood is otherwise normal; so is her physical examination. Question What should be administered in the evenings in order to improve this patient's sleep problems and nighttime confusion?

Risperidone (tx for "sundowning" in AD)

Of the following Personality Disorders, a person with which disorder is likely to be most resistant to therapy? Borderline Personality Disorder Narcissistic Personality Disorder Dependent Personality Disorder Histrionic Personality Disorder Schizoid Personality Disorder

Schizoid Personality Disorder Essential features of Schizoid Personality Disorder is a detachment from social relationships, a restricted range of emotions, lack of warmth, and an indifference to others' opinions of them. Because of these traits, schizoids typically do not seek treatment and probably lack the motivation to benefit from psychotherapy. A person with Borderline Personality Disorder (BPD) is apt to regress if they perceive therapy is going well as a means of sabotaging a goal. Those with BPD may develop patterns of undermining successes just at the moment they may be realized. Narcissistic Personality Disorder is marked by grandiose, egotistical, and arrogant behaviors. The narcissist would love therapy because it would allow him to be the center of attention. A person with a Dependent Personality Disorder would display dependent behaviors such as clinging, inability to make decisions without excessive advice seeking from others, and so on. This person would benefit a great deal from therapy, although the therapist's job would be very difficult. Histrionics are typically over-dramatic and enjoy calling attention to themselves. They would not be interested in traditional psychoanalysis but might benefit from confidence and esteem building.

A 23-year-old woman comes to see you because her mom told her she should see a doctor for "her problem". When asked about "her problem", the woman states that she just does not like being around people and prefers to be alone. Her mom does not believe that this is normal. When asked how long she has felt this way, the client states that she started progressively isolating herself more and more about 6 years ago. She does not have any hobbies, and she prefers to just sit in her room or maybe put together a puzzle. She wishes that she could just disappear sometimes so that she would not have to talk to anyone. The woman has no history of alcohol or drug use, and she has never had sexual intercourse. She states that she has no desire to have intimacy with another person. She denies having any delusions or hallucinations. She has never experienced mania or depression. She states that she does not really feel anything at all. The client expresses very little emotion throughout the course of the interview. Question What is the most likely diagnosis?

Schizoid personality disorder This client expresses a lack of desire for social relationships, a preference for solitary activities, little interest in most activities or intimacy, and has demonstrated a flattened affect. These symptoms are all characteristic of schizoid personality disorder. Furthermore, this woman's symptoms had an onset around the beginning of adulthood, and she is not experiencing any psychotic symptoms, which further lends support for this diagnosis. Schizoid personality disorder can be differentiated from autism spectrum disorder by the age of onset. In autism spectrum disorder, symptoms must have been present before the age of 3. It can be differentiated from paranoid personality disorder by the lack of suspicious or paranoid ideation. In avoidant personality disorder, a person isolates himself or herself due to a fear of social interactions; in contrast, people with schizoid personality disorder isolate themselves from social interactions due to a lack of desire.

A 51-year-old woman is brought by her concerned family. She presents with a long history of eccentric behavior and reclusiveness. She always dresses in black and is very superstitious, rarely letting anyone into her home or engaging in social activities. On exam, she is slightly disheveled and speaks in grandiose metaphors, but has no active delusions or hallucinations. Question What is the most likely diagnosis?

Schizotypal personality disorder

A 70-year-old woman is referred to you to be seen in a nursing facility. She was recently evicted by an apartment building for odd and eccentric behavior. The previous property manager reports that the patient would wear winter clothing in the summer time, would stay off to herself and never socialize with other peers in the building, and would not want people to enter her apartment for any reason. At the nursing home, she presents with the same behavior, and even talks of communicating with aliens and going to another planet. She is alert and oriented, has no history of serious medical problems, other than arthritis and hypertension and scores perfectly on the mental status exam. Question What is the most likely diagnosis?

Schizotypal personality disorder Paranoid personality characteristics consists of the following: Unjustified distrust and suspicion of others, the inability to confide in others and the ability to carry grudges and resentment of peers or colleagues for no founded reason. They appear cold, calculating, and see suspicious meanings in everything. They usually have occupational difficulties and only do well if they have a solitary job. Schizoid personality characteristics include being lifelong loners, restricted emotional range, unsociable behavior, appearing cold, seclusive, enjoy few activities, and usually do not marry or have romantic relationships. They often daydream and carry an unusual attachment to animals, but not people. They are unaffected by praise or criticism and have no interest in sexual behavior. Schizotypal personality characteristics include behavior that is influenced by magical thinking, telepathy, aliens, unusual perceptions or bodily delusions, odd behavior and appearance, extreme anxiety in social situations, and affect that is blunted or inappropriate to the topic at hand.

A 25-year-old man has a long history of criminal behavior, and he has been in and out of prison for assaultive behavior, theft, armed robbery, and sexual assault. He shows no remorse for his behavior and states that he is the real victim. He also shows little emotion regarding his family history, and he prefers to brag about his sexual exploits. The patient denies a history of mood disorders or schizophrenia. Question What disorder commonly co-occurs with this patient's symptoms?

Substance use disorder (antisocial)

A 68-year-old man presents with his wife of 38 years. She states that since her husband retired 3 years ago, he just sits home and drinks an entire bottle of either Jack Daniels or Chivas Regal every day, and he eats very little. She also mentions that she thinks he may have broken his hip in a fall last weekend, but he is too drunk to feel pain. Question In addition to detoxification with a benzodiazepine prior to repair of his hip fracture, what is needed to prevent neurologic sequelae?

Thiamine

A 48-year-old woman presents with vision difficulties. She is well known to the department due to frequent visits for alcohol-related complaints. She cannot give any history as to her recent whereabouts. Physical exam reveals a disheveled woman who appears older than her stated age. Vital signs show: BP 155/80 mm Hg; pulse 100; respiration 16; T: 37.5° C (99.5° F). HEENT exam reveals a marked aroma of alcohol. Heart and lung exams are unremarkable. Abdominal exam shows mild, diffuse tenderness without rebound or guarding. Neurologic exam reveals lateral rectus palsy, marked bilateral nystagmus, and an ataxic gait. Short-term memory is poor. Question What should the initial treatment be?

Thiamine (Wernicke's Encephalopathy)

A slightly overweight 15-year-old girl presents for evaluation because of concerns about an eating disorder. Her family recently moved, and she feels overwhelmed and stressed adapting to a new school environment. She misses her old friends and feels she has difficulty making new friends because she is overweight. According to her parents, she continues to consume large amounts of food even though her weight is not changing. Question What additional finding would best distinguish between this patient's two most likely diagnoses?

Using laxatives to lose calories (binge-eating disorder vs bulimia)

A 57-year-old man presents to the emergency department complaining of difficulty with his vision. He is well known to the department due to frequent visits for alcohol-related complaints. He cannot give any history as to his recent whereabouts. Physical exam reveals a disheveled male patient who appears older than his stated age. Vital signs show: BP 155/80 mm Hg; pulse 100; respiration 16; temp 37.5°C (99.5°F). HEENT exam reveals a marked aroma of alcohol. Heart and lung exams are unremarkable. Abdominal exam shows mild, diffuse tenderness without re-bound or guarding. Neurologic exam reveals lateral rectus palsy, marked bilateral nystagmus, and an ataxic gait. Short-term memory is poor. Question What is the etiology of this patient's complaints.

Wernicke's encephalopathy

A 58-year-old man presents for further evaluation after being brought in by the local police because of an unstable gait and disheveled appearance; his breathalyzer test is 0.08 g/dL. On exam, the man is gaunt and clearly malnourished; he is disoriented and confused and unable to respond to questioning. His right elbow and knee are bruised, but he appears to have no other significant injuries; however, his eye movements are uncoordinated and bounce from side to side. He is unable to fixate his gaze. Vital signs are as follows: Blood pressure89/58 mm HgPulse109Temperature35.7°C Question What diagnosis is most consistent with the patient's signs and symptoms?

Wernicke's encephalopathy Ethanol has myriad effects on the central nervous system. Wernicke's encephalopathy is a medical condition resulting from thiamin deficiency. It is most commonly observed in individuals who drink heavily but consume little food. Since the metabolism of alcohol requires this nutrient, the body's stores of thiamin can become depleted under these conditions. The signs and symptoms of Wernicke's encephalopathy are similar to acute alcohol intoxication: disorientation, confusion, indifference, inattentiveness, and incoordination/gait ataxia. The feature that distinguishes Wernicke's encephalopathy, however, is ophthalmoplegia, which reverses rapidly following intravenous infusion of thiamin. Patients with Wernicke's encephalopathy who remain untreated often develop permanently impaired past and new memory but preservation of long-term memory and other cognitive skills, a condition known as Korsakoff's syndrome.

You see a woman in her mid-40s for the first time in your office. In the course of your history taking, you learn that as a child, she witnessed her paternal grandfather slowly die from cirrhosis of the liver, and as an adolescent, she saw her father's drinking ruin her parents' marriage. While in college, she learned that a couple of glasses of wine could ease the pain of what she came to accept as a chronic sense of anxiety. The patient learned to limit her drinking to 2 glasses on any given day, with no exceptions. Despite her vigilance, however, on 2 occasions the patient did drink more than her allotment during periods of extreme stress; they were an abortion and her mother's diagnosis of Alzheimer's disease. Both incidences landed her in traffic court less than a year apart, with charges of driving while intoxicated. The patient noted that both times she drank to intoxication and smoked several cigarettes as well. Question What is the most likely diagnosis?

alcohol abuse

A 22-year-old man presents to establish care. He states that he was beaten up by his parents as a child, and he was sent to a series of foster homes because he ran away from his home several times. His schooling was rare; he was frequently in trouble for truancy, vandalism, starting fights, and stealing. He dropped out of school at age 15; during that year, he was arrested for car theft and driving while drunk. He has not worked at any job for more than 4 months and has had frequent changes of address because he did not pay rent or meet other financial obligations. He boasts about being the father of two children by two different women, but he has not provided any support or made any contact with either of them since their pregnancies. He has used several aliases. He has no history of a psychotic break. His IQ testing is normal. Question What is the most likely diagnosis?

antisocial personality disorder

A 25-year-old man rarely goes out, and his mother is pressuring him to get a girlfriend. He reveals that he has never felt comfortable around other people because he feels too much pressure to "perform." He fears rejection. Question What type of personality disorder does he exhibit?

avoidant

A 26-year-old woman is having marital difficulties with her husband; she is a flight attendant, and he is a pilot for another airline. The woman dropped out of college during her senior year, occasionally smokes marijuana, and has had breast implants. Frequently, she and her husband get into violent fights over little things at home, primarily his not being attentive enough to her. At these times, she engages in self-mutilating activities. The next morning, she awakens in an amorous mood and wants to make love, feeling that he is the most wonderful man on this earth. More than once, she has made a suicidal gesture when she thought her husband would leave her. She constantly fears that her husband is going to abandon her someday. When stressed, she becomes paranoid and has a feeling of being able to step outside her body and watch herself. She also has a terrible sense of emptiness and a tendency for binge eating. She is very impulsive and often goes off on shopping sprees. Question What is the most likely diagnosis?

borderline

A 48-year-old woman has a history of poor interpersonal relationships, chronic headaches, and previous suicide attempts. She has an extremely demanding attitude and often directs her anger towards others. Question What personality disorder is the most likely diagnosis?

borderline The clinical picture is suggestive of a borderline personality disorder. These patients typically present with the following clinical findings: they are impulsive, have unstable interpersonal relationships, are suffused with anger and fear, lack self-control, are suicidal, and demonstrate aggressive behavior. Narcissistic personalities are exhibitionists, grandiose, preoccupied with power, and have poor social interactions. Avoidant personalities fear rejection, have low self-esteem, and hyper-react to rejection and failure. Histrionic personalities are dependent, immature, seductive, vain, and egocentric. Antisocial personalities are described as selfish, callous, impulsive, associated with having legal problems, and are promiscuous.

A 34-year-old man presents for the first time in your office complaining of episodic restlessness, nervousness, sleeplessness, tirelessness, increased urination, and gastrointestinal upset. A quick physical reveals an irregular heartbeat and he appears to have psychomotor agitation. You observe that his face is quite red and his speech is rapid. The patient complains that he is unable to function well at school when these "attacks" occur, and they are increasing in frequency, now that he is studying for exams. Interview and history reveal that the patient has no personal or known family history of mental illness, and physical examination is unremarkable except for the current symptoms. The patient says that he has been under a lot of pressure lately because he is working long shifts in a local café, and he studies most of the rest of the time for upcoming doctoral comprehensive exams. He drinks coffee for most of his shift, and he eats chocolate covered espresso beans to stay alert while he studies. Question What is the most likely diagnosis?

caffeine intoxication

A 24-year-old male is brought to the emergency room. He is highly agitated and will not sit still. His speech is rambling and is dotted with paranoid delusions. About 1 hour later, the patient's excited speech slows and he quietly mutters to himself before falling asleep. The patient is most likely suffering from:

cocaine intoxication

A 30-year-old man has been brought into the ER in a state of euphoria, but showing obvious impairment in judgment. He was walking on the railing of a local bridge when his friends managed to get him down and bring him in. Lately, he has been neglecting his work, going in late, and feeling unappreciated. His pupils are dilated, his BP is 150/96 mm Hg, diaphoretic, his clothes are too loose to be stylish, and he is perspiring heavily. Patient also complains of chest pain. The manner in which he relates to you is somewhat condescending and he is grandiose in his thinking. There is no history of psychiatric illness and no medical illness that you can detect. Question What is the most likely diagnosis?

cocaine intoxication Cocaine intoxication presents with sudden onset of chest pain associated with a racing heart, diaphoresis and dilated pupils. Patients may appear agitated or present with pressure speech. Bipolar disorder is characterized by depression and mania. Adjustment disorder occurs within 3 months of a stressor and is characterized by changes in mood and behavior that affect daily living. Alcohol withdrawal presents with feelings of anxiety, increased heart rate and blood pressure, hallucinations and possible seizures. Intermittent explosive disorder, which can be diagnosed in children as young as six, is characterized by a wide variety of aggressive outbursts.

A 55-year-old man is agitated and anxious and disoriented; he appears to be responding to visual hallucinations. According to his partner, the patient has been very moody lately and he has had a lot of difficulty sleeping. When he does sleep, according to the partner, the patient has frightening dreams. Examination reveals a man in significant distress: tremor, sweating, tachycardia, hyperventilation, and slow response to questions. The patient's nasal septum is perforated. Question You perform a urine toxicology screen, and proceed to treat the patient for which condition while you await the results?

cocaine use The signs and symptoms of cocaine use are often very similar to that of amphetamine use. Of the signs and symptoms noted above, all would be consistent with amphetamine use except perforation of the nasal septum, which suggests that the patient used his drug intranasally. Hypnotic use is characterized by the misuse of drugs whose therapeutic effect is to induce and maintain sleep. Often, drugs in this category are misused because of their tendency to produce euphoria or to counter restlessness and other unintended effects of other drugs. The criteria for polysubstance use is the use of three or more substances at once, none of which predominates. The criteria the substances are used despite adverse effects or in situations in which use is physically hazardous must be met as a group to make a diagnosis of polysubstance use. Sedative use is characterized by either episodic or chronic use of sedating drugs to reduce excitement and induce quiet without the drowsiness of other central nervous system depressants. Examples of the misuse of sedatives include using the drug despite adverse effects, in situations where it is physically hazardous to do so, or to experience its side effects rather than its therapeutic effects. Alcohol use is characterized by regular daily intake of large amounts of alcohol, regular heavy drinking limited to weekends, or binge periods of heavy drinking interspersed with long periods of sobriety. Alcohol is a central nervous system depressant.

A patient with an amnestic syndrome is approached by a team of physicians, residents, and students. The resident, who has never met the patient, says, "Hello Mr. Smith, do you remember me?" The patient responds with a story about the 2 of them meeting in the hospital cafeteria the day before. Question What is this syndrome called?

confabulation Anterograde amnesia is memory loss of events that occur after a specific CNS insult, and retrograde memory loss is memory loss of events that occurred before a CNS insult.

A 24-year-old man presents with rambling speech that is dotted with paranoid delusions. He is highly agitated and will not sit still. About 1 hour later, the patient's excited speech slows, and he quietly mutters to himself before falling asleep. On Examination: Temp- 37°C, PR- 108/min, BP- 140/70 mm Hg, and RR- 22/min. General examination reveals irregular skin flushing over the trunk. Other system examination is normal. Question What helps in differentiating amphetamine from cocaine intoxication?

duration of the high Typically, the effects of a single dose of amphetamines lasts 4 to 6 hours, and those from cocaine rarely last more than 2 hours.

You have been working with a 14-year-old Caucasian boy. He was referred to you after being arrested for the third time. His offenses range from petty theft to illicit drug use. His most current offense involved stealing and going joyriding while under the influence of alcohol. During the assessment interview, he was cooperative, but minimized his crimes. He stated that these are things that all boys his age do, he just got caught. Question What is the most effective way to treat this individual?

family therapy (best for children with behavior problems)

A 16-year-old boy presents with trouble breathing, excessive coughing, headache, abdominal pain, and vomiting. A toxicology screening returns negative. The patient is not suffering from pneumonia or bronchitis; he has no history of asthma. Question Based on these findings and patient history, what is the most likely diagnosis?

inhalant use Inhalants are typically not part of a routine toxicology screening, and given the limited and nonspecific availability of symptomology for inhalant use, this diagnosis might only be made through the process of elimination. A long-term inhalant user might suffer from both CNS and PNS damage, in addition to suffering damage to muscles, the liver, and other organs.

You are the ER resident on duty and the paramedics call in to report that they are bringing an "elderly" African American male who appears to be suffering from alcohol hallucinosis. Which of the following symptoms most closely describes this syndrome? it manifests itself with paranoid ideation 2 it usually affects people who have reached middle age (>40 years of age) 3 it usually lasts for a brief period of time 4 noticeable generally within a few hours after the person stops drinking 5 it usually manifests itself with seizure disorder ranging from petit to grand mal

it manifests itself with paranoid ideation

You have been working with a 40-year-old man. When he is with his partner, he reports that he has no difficulty achieving an erection, but whenever they are about to begin sexual intercourse he cannot seem to maintain the erection. Question What phase of the sexual response cycle is this client having difficulty with?

plateau phase

A 43-year-old woman presents for the first time in your office. Chief complaints are related to the use of alcohol, "uppers," benzodiazepines and, on occasion, marijuana. The patient says that she has been using all of these substances on and off for most of her adult life and that she has never used any of them enough to suffer from withdrawal when they are not used. She uses them mostly in social and recreational situations with others, and always in combination. For example, when she gets high on uppers, she feels less anxious if she uses benzodiazepines to temper the high. She never uses benzodiazepines unless she has had a drink or two first. While she understands the potential dangers of mixing these drugs, the patient states, "When I use one, I have no control over what else I use". The patient has come to your office today for an assessment so that she may enroll in a vocational rehabilitation program. Question After taking a thorough history, which is essentially unremarkable except for the information above, you are most likely to recommend treatment for what?

polysubstance abuse

A diagnosis of Schizoid Personality Disorder instead of Schizophrenia would be made in the absence of:

psychosis

Which of the following deteriorates first in Alzheimer disease?

recent memory

A 15-year-old girl has been dieting for 6 months and has lost over 30 pounds. She tells you that she still feels fat. She is afraid to eat for fear of becoming obese. Her last menstrual period was 3 months ago. On physical exam, the patient is cachectic and slightly pale. Her heart rate is 50 beats/minute, her blood pressure is 90/60 mm Hg, and her temperature is 95.5°F. Her weight is 92 lb and her height is 5'6". Question What laboratory value is most likely present?

reduced LH

A 20-year-old female college student aspiring to become a professional ballet dancer reports to her primary care physician concern about her ability to have children since her menarche has not started yet. She is 5'8" tall and weighs 100 lb. She is overly concerned about being overweight and diets almost every day of the week. She says that she has been recently vomiting frequently. She is fatigued and looks pale. Blood tests reveal a slight hypokalemia. Question What is the best initial step in management of this patient?

restore electrolytes

A woman in her early 30's is seen in your office for the first time. She gave birth to a healthy male infant 8 months ago, and since that time she has experienced excessive anxiety that she relates to being a new mother. More recently, her mood has been quite low and her husband has expressed concern that she is depressed. The patient says that her son is very active and, as a means of keeping up, she started using diet pills. They boost her energy as she had hoped they would, but she also finds that once she is energized she often has to bring herself down so that she may relax in preparation for a good night's sleep. To do this, the patient says that she routinely takes diazepam, 20 mg. The patient admits that she abuses amphetamines and that she needs to find other ways to cope with the demands of being a mother. The history and physical reveals no other remarkable information. Question In addition to the likelihood of amphetamine abuse, what is the patient exhibiting signs of?

sedative abuse Sedative abuse is characterized by either episodic or chronic use of drugs such as diazepam, which reduce excitement and induce quiet without the drowsiness of other central nervous system depressants. Examples of the abusive use of sedatives include using the drug despite adverse effects in situations where it is physically hazardous to do so or to experience its side effects rather than its therapeutic effects. In this case, the patient admits to using diazepam as a means of tempering the effects of amphetamines. This meets the criteria for using the drug to exploit its side effect profile. It is also possible that using amphetamines and diazepam regularly could put her baby in danger, which would also be consistent with sedative abuse. It is common to abuse sedatives in combination with other drugs. Opioid abuse is characterized by the abuse of opiates and opioids, including morphine, hydromorphone, and heroin. These drugs are often prescribed to treat pain but also have euphoric effects in some people. The criteria for polysubstance abuse is the use of three or more substances at once, none of which predominates. The criteria for abuse is that the substances are used despite adverse effects, or in situations in which use is physically hazardous, must be met as a group to make a diagnosis of polysubstance abuse.

A 2-year-old boy presents because of his mother's concerns about his sleep. She notes that he is easy to put to sleep, and he has a regular bedtime of 7:30 PM. Most nights for the past 3 weeks, about 90 minutes after being put to sleep, he begins to thrash violently in bed, sometimes letting out bloodcurdling screams. He has his eyes open and seems to be talking, but he does not respond to either parent when spoken to. He sweats a lot. The episodes last about 15 minutes, and he then goes back to sleep; he seems fine in the morning. He naps for about an hour in the morning and an hour in the afternoon, but these episodes do not occur with naps. Question What is the most likely diagnosis?

sleep terrors

A 33-year-old woman presents after being found unresponsive in her bedroom. She has a past medical history of depression, and her mother found an empty bottle of amitriptyline by her bedside. Otherwise, the patient has no other medical or surgical history. She is a non-smoker and does not drink alcohol. On physical exam, pulse 138/minute, blood pressure 80/60 mm Hg, temperature 101.2°F (38.4°C), respirations 6/minute. Her heart sounds are normal, and she has thready pulses. Her breath sounds are normal but with shallow effort. The abdomen is soft and non-tender. Neurologically, she moves her limbs from painful stimuli. Her skin is flushed; there are no needle marks. Her chest X-ray is normal, and the electrocardiogram demonstrates a wide complex tachycardia without ectopy. The patient is intubated and hyperventilated. Question What is the next step in this patient's management?

sodium bicarbonate Sodium bicarbonate is the drug of choice for the treatment of ventricular dysrhythmias or hypotension secondary to TCA poisoning.


संबंधित स्टडी सेट्स

BUSN for The Common Good Final Review Questions:

View Set

Chapter 35 - The Digestive System

View Set

Exam 2 Pharmacology (Cardiac Medications)

View Set